LSAT and Law School Admissions Forum

Get expert LSAT preparation and law school admissions advice from PowerScore Test Preparation.

User avatar
 Dave Killoran
PowerScore Staff
  • PowerScore Staff
  • Posts: 5849
  • Joined: Mar 25, 2011
|
#87487
Complete Question Explanation
(The complete setup for this game can be found here: lsat/viewtopic.php?f=149&t=16450)

The correct answer choice is (E)

The best strategy on List questions is to use the individual rules of the game to eliminate incorrect answer choices:

Answer choice (A) can be eliminated because according to the last rule two of the monuments were begun in 601.

Answer choice (B) can be eliminated because according to the second rule H was begun no earlier than 604.

Answer choice (C) can be eliminated because according to the first rule L was begun in an earlier year than F.

Answer choice (D) can be eliminated because according to the third rule M was begun earlier than 604.

Answer choice (E) is thus proven correct by process of elimination.

Get the most out of your LSAT Prep Plus subscription.

Analyze and track your performance with our Testing and Analytics Package.